Principles of Real Analysis - (Chapter 4 Infinite Series)
Principles of Real Analysis - (Chapter 4 Infinite Series)
CHAPTER 4
Infinite Series
4.1 INTRODUCTION
In this chapter we shall discuss the techniques of testing the behaviour of infinite series as regards
convergence. The most important technique for series, all of whose terms are of the same sign (all positive
or all negative), is to compare the given series with another suitably chosen series with known behaviour.
So, first of all, comparison tests are discussed, and then some special tests for convergence are considered.
Leibnitz’s test for alternating series, Abel’s and Dirichlet’s tests for arbitrary term series, and Dirichlet’s
and Riemann’s theorems on rearrangement of terms will be discussed in detail towards the end.
4.1.1 A series is the sum of the terms of a sequence. Thus if u1, u2, u3, ... is a sequence then the sum
∞
u1 + u2 + u3 + ... of all the terms is called an infinite series and is denoted by ∑ un or simply by ∑ un .
n =1
Evidently we cannot just add up all the infinite number of terms of the series in the ordinary way and
in fact it is not obvious that this kind of sum has any meaning. We thus start by associating with the given
series, a sequence {Sn}, where Sn denotes the sum of the first n terms of the series. Thus
S n = u1 + u2 + ... + un ,
The sequence {Sn} is called the sequence of partial sums of the series and the partial sums, S1 = u1,
S2 = u1 + u2, S3 = u1 + u2 + u3, and so on, may be regarded as approximations to the full infinite sum
∞
∑ un of the series. If the sequence {Sn} of partial sums converges, then the series is regarded as
n =1
convergent, and lim Sn is said to be the sum of the series. If however, {Sn} does not tend to a limit, we
must take it that the sum of the infinite series does not exist. We express this fact by saying that the series
does not converge. In fact an infinite series is said to converge, diverge or oscillate according as its
sequence of partial sums {Sn} converges, diverges or oscillates.
4.1.2 A Necessary Condition for Convergence
Copyright © 2004. New Academic Science. All rights reserved.
Theorem 4.1. A necessary condition for convergence of an infinite series ∑ un is that lim un = 0.
n ®¥
Now,
un = Sn − Sn −1 , n > 1
=s–s=0
Hence for a convergent series
lim un = 0
In other words, a series cannot converge if its nth term does not tend to zero,
Notes 1. It must be clearly understood that lim un = 0 does not prove that a series in convergent, for there exist series
which do not converge even though lim un = 0. See Example 4.2 below.
2. However, lim un ≠ 0 proves that the series does not converge, see Example 4.1 below:
Theorem 4.2. A series ∑ un converges iff to each ε > 0, there exists a positive integer m such that
Sn + p − Sn < ε , ∀ n ≥ m and p ≥ 1
Copyright © 2004. New Academic Science. All rights reserved.
or
1
Example 4.2. Show that the series ∑ does not converge.
n
Solution. Suppose, if possible, the series converges.
1
Therefore, for any given ε say, , ∃ a positive integer m, such that
4
1 1 1
+ + ... + < ε , ∀ n ≥ m and p ≥ 1
n+1 n+2 n+ p
In particular, if n = m and p = m, we get
1 1 1 1 1 1
+ + ... + = + + ... +
n+1 n+2 n+ p m+1 m+2 2m
1 1
> m⋅ = >ε.
2m 2
Thus, there is a contradiction. Hence, the given series does not converge.
1
It may also be seen that lim un = lim = 0 even though the series does not converge.
n
4.1.4 Some Preliminary Theorems
Theorem 4.3. If å un = u, then å cun = cu , where c is any number independent of n.
The result follows at once from the identity
n n
∑ cur = c ∑ ur
r =1 r =1
n n
Let Sn′ = ∑ ur and Sn = ∑ ur .
r=0 r =1
Clearly
S n′ = u0 + S n
∴ By letting n tends to infinity
∞
∑ un = u0 + u
Copyright © 2004. New Academic Science. All rights reserved.
n=0
∞ ∞ ∞ ∞
It is also clear that if the series ∑ un is divergent, the changed series ∑ cun , ∑ un or ∑ un is
n =1 n =1 n=0 n=2
also divergent.
Hence, the behaviour of a series as regards convergence is not altered by
(i) the alteration, addition or omission of a finite number of terms; or
(ii) multiplication of all the terms by a finite number other than zero.
Theorem 4.5. (Convergent series may be added or subtracted term by term). If ∑ un = u and ∑ vn = v ,
then ∑ wn = u ± v , where wn = un ± vn , for all n.
The result follows from the identity
n n
b g n
∑ wr = ∑ ur ± v r = ∑ ur ± ∑ v r
r =1 r =1 r=1
n
r =1
shown later that the brackets may be inserted or removed without affecting convergence.
2. The theorem may as well be proved by the following alternative argument.
Since the given series converges to u, its sequence {Sn} of partial sums will also converge to u. Therefore,
the sequence {σ n } of partial sums of ∑ vn , being a subsequence of {Sn} will also converge to u.
3. The theorem may be restated, “A series obtained from a given convergent series by a grouping of terms
converges to the same limit”.
By grouping we simply mean the placing of brackets or associating the terms of the series without
changing the order of the terms.
Remarks 1. The sequence of partial sums of a series with negative terms can be shown to be monotonic decreasing and
hence a series with negative terms converges iff the sequence of its partial sums is bounded below.
2. It may similarly be seen that a series of negative terms can either converge, or diverge to – ∞.
3. A series of positive terms can either converge, or diverge to + ∞. But a series with arbitrary terms can have
five possible behaviours depending upon the behaviour of the sequence of its partial sums.
Case I. 0 ≤ r < 1.
Let {Sn} be the sequence of its partial sums, so that
Sn = 1 + r + r2 + ... + rn – 1
i.e.,
1 − rn 1 rn 1
Sn =
= − ≤ , ∀ n,
1−r 1− r 1− r 1− r
so that {Sn} is bounded above and hence the series converges.
Copyright © 2004. New Academic Science. All rights reserved.
Case II. When r = 1, Sn = n, so that {Sn} is not bounded above and hence the series diverges to + ∞.
Case III. When r > 1, every term of Sn after the first is greater than 1, so that
S n > n, ∀ n .
Therefore, the sequence {Sn} is not bounded above and consequently the given series diverges
to + ∞.
Hence, the given series converges if r < 1 and diverges if r ≥ 1.
4 p
+ p + p + p <
5 6 7 4 p
= p−1
4
=
H2 Kp −1
1 1 1 8 1
=G
F 1 IJ 3
8 p
+ p + ... + p <
9
... ... ... ... ...
15 8 p
= p −1
8
... ...
H2 K
... ...
p −1
...
... ... ... ... ... ... ... ... ... ...
1 1 1 2n + 1 − 2n 1 F 1 IJ
=G
n
d2 i d2
n p
+
n
+1 i p
+ ... +
d2 n +1
−1 i p
<
d2 i n p
=
d2 i
n p −1 H2 K
p −1
Adding,
1 FG 1 IJ 2
FG 1 IJ
n
S2 n + 1 − 1 < 1 +
2p −1
+
H2 K p −1
+ ... +
H2 p −1 K
F 1 IJ n +1 LM1 − FG 1 IJ OP
n+1
1−G
MN H 2 K PQ
2p−1
=
H2 K p −1
=
p−1
1−
1 2 p− 1 − 1
p −1
2
p −1
2
, ∀n < p −1
2 −1
Copyright © 2004. New Academic Science. All rights reserved.
2p −1
∴ Sn < S2 n < S2 n + 1 − 1 < .
2p −1 − 1
2p −1
Since for a given p, is a fixed number, therefore, the sequence {Sn} of partial sums of the
p−1
2 −1
given positive term series is bounded above and hence the series converges for p > 1.
Case II. When p ≤ 1.
We know, if n is any positive integer and p ≤ 1, then
1 1
np ≤ n ⇒ ≥
np n
1 1 1
∴ p
1+
≥1+ >
2 2 2
1 1 1 1 2 1
+ ≥ + > =
3p 4 p 3 4 4 2
1 1 1 1 1 1 1 1 4 1
p
+ p + p + p ≥ + + + > =
5 6 7 8 5 6 7 8 8 2
1 1 1 1 1 1 8 1
p
+ p + ... + p ≥ + + ... + > =
9 10 16 9 10 16 16 2
... ... ... ... ... ... ... ... ...
... ... ... ... ... ... ... ... ...
1 1 2 1 1 1 2 m − 2m − 1 1
+ + ... + ≥ + + ... + > =
d2 m−1
+1 i d2
p
m −1
+2 i p
d2 i
m
p
2m − 1 + 1 2m − 1 + 2 2m 2m 2
Adding,
m
. S2 m >
2
We shall now show that {Sn} is not bounded above.
If G be any number, however large, then ∃ m ∈ N such that
m
>G
2
Let n > 2m,
∴ Sn > S2m > G
Thus, the sequence {Sn} of partial sums of the given series is not bounded above, and hence the
series diverges for p ≤ 1.
Copyright © 2004. New Academic Science. All rights reserved.
1
Thus, the given series ∑ converges iff p > 1.
np
terms of one series will be compared to the ratio of the corresponding consecutive terms of the second
series.
4.3.1 Comparison Test (First type)
I. If ∑ un and ∑ vn are two positive term series, and k ¹ 0, a fixed positive real number (independent
of n) and there exists a positive integer m such that un ≤ kvn , ∀ n ≥ m, then
(i) ∑ un is convergent, if ∑ vn is convergent, and
(ii) ∑ vn is divergent, if ∑ un is divergent.
Let n ≥ m, Sn = u1 + u2 + ... + un, and tn = v1 + v2 + ... + vn.
Now " n ³ m, we have
Sn − Sm = um + 1 + um + 2 + ... + un
d i b
≤ k vm + 1 + vm + 2 + ... + vn = k t n – t m g
or b
Sn ≤ kt n + Sm − kt m g
⇒ S n £ kt n + h ...(1)
where h = Sm – ktm, is a finite quantity.
(i) If ∑ vn is convergent, then the sequence {tn} of its partial sums is bounded above, so that ∃
a number B such that
tn ≤ B, ∀ n .
So from (1), we get
Sn ≤ kB + h, for all n ≥ m,
⇒ the sequence {Sn} is bounded above.
⇒ å un is convergent.
(ii) If ∑ un is divergent, then the sequence {Sn} of its partial sums is not bounded above, so that
if G be any number, however, large, ∃ a positive integer m0 such that
Sn > G " n ³ m0 .
Thus from (1), ∀ n ≥ max ( m, m0 ),
1
tn ≥ ( G − h), k ≠ 0
k
⇒ the sequence {tn} is unbounded
⇒ åvn is divergent.
Copyright © 2004. New Academic Science. All rights reserved.
FG u IJ = l , where l is a
Hv K
n
II. Limit Form. If å un and å vn are two positive term series such that lim
n →∞
n
non-zero finite number, then the two series converge or diverge together.
Evidently l > 0.
Let ε be a positive number such that l − ε > 0.
un
Since lim = l , therefore ∃ a positive integer m such that
n→∞ vn
un
- l < A,
vn "n³m
un
⇒ l-A< < 1 + A, "n³m
vn
⇒ bl - A g v n b g
< un < 1 + A v n , "n³m ...(1)
Now, if ∑ vn is convergent, then from (1)
un < (1 + A ) vn , "n³m
so that by Test I, ∑ un is convergent.
Again, if ∑ vn is divergent, then from (1)
b
un > 1 - A vn , g "n³m
so that by Test I, ∑ un is divergent.
Similarly, we may show that ∑ vn converges or diverges with ∑ un . Hence, the two series behave
alike.
4.3.2 Comparison Test (Second type)
III. If ∑ un and ∑ vn are two positive term series and ∃ a positive integer m such that
un v
≥ n , ∀ n ≥ m,
un + 1 v n + 1
then (i) ∑ un is convergent, if ∑ vn is convergent, and (ii) ∑ vn is divergent, if ∑ un is divergent.
Let m be a positive integer and Sn = u1 + u2 + ... + un
and tn = v1 + v2 + ... + vn .
For n ≥ m, we have
um u um + 1 un − 1 v vm + 1 vn − 1 v m
= m ⋅ ... ≥ m ⋅ ... =
un um + 1 um + 2 un vm + 1 vm + 2 vn vn
um
⇒ un £ vn .
vm
Since m is a fixed positive integer, therefore um/vm is fixed number, say k. Thus ∀ n ≥ m, we have
Copyright © 2004. New Academic Science. All rights reserved.
un ≤ kv n
Hence by Test I, ∑ un converges if ∑ vn converges and ∑ vn diverges if ∑ un diverges.
Notes 1. For practical purposes, Test II is very useful and can be easily applied.
2. For a successful application of the comparison test, we first make an estimate of the magnitude of the general
term un of the given series, and then select the auxiliary series ∑ v n of such a magnitude that
lim( un /vn ) = l ≠ 0, ∞, or in other words un ~ vn . Thus for large values of n,
nr
n 3 + 1 ± ~ ± n 3/2 , ~ ± nr - s
(1 + n ) s
1 1
sin ±~±
n n
3. It will help to remember that for large n, e an >> n b >> ( log n) c , where a, b, c are positive numbers.
+ + ... + + ...
blog 2g blog 3g
p p
blog ng p
= 0, therefore
n →∞ n
blog ng p
< n, ∀ n > 1
1 1
> , "n>1
∴
blog ng p
n
Let us compare the given series with the divergent series
1 1 1
+ + + ...
2 3 4
Since each term of the given series is greater than the corresponding term of the divergent series,
therefore, the given series diverges.
Example 4.5. Show that the series
1⋅ 2 3⋅ 4 5⋅ 6
2 2
+ 2 2 + 2 2 + ..., is convergent
3 ⋅4 5 ⋅6 7 ⋅8
Solution. Let us denote the given series by ∑ un , where
b2n − 1g b2ng , FG ~ 1 IJ .
b2n + 1g b2n + 2g H n K
un = 2 2 2
Example 4.7. Test for convergence of the series whose nth term is
{(n3 + 1)1/3 – n}
Solution. Let un = n 3 + 1 d i −n
1 /3
|RF 1 I
= n SG1 + J
1/3
|UV
|TH n K 3
−1
W|
RS 1 UV 1 FG ~ 1 IJ
=n
T 3n 3
+ ... =
W 3n 2
+ ...
H nK 2
1
Let ∑ v n = ∑
n2
Now
un 1
lim =
n →∞ vn 3
Therefore, the two series converge and diverge together.
Since, ∑ vn converges, therefore, the given series also converges.
1
Example 4.8. Test the convergence of the series ∑ 1 + 1 /n
.
n
1 1
Solution. Let un = ∑ and vn =
n1 + 1/n n
un 1
Now = lim 1/ n = 1 lim
vn n →∞ n n →∞
EXERCISE
1 3 5 7
1. + + + + ...
1⋅ 2 ⋅ 3 2 ⋅ 3⋅ 4 3 ⋅ 4 ⋅ 5 4 ⋅ 5 ⋅ 6
1 1 1
2. + + + ...
1⋅ 2 2⋅3 3⋅4
1 3 5 7
3. + + + + ...
4 ⋅ 6 6 ⋅ 8 8 ⋅ 10 10 ⋅ 12
n+1
4. ∑
np
1
5. ∑
n + n+1
Copyright © 2004. New Academic Science. All rights reserved.
6. ∑ FH n4 + 1 − n4 − 1 IK [Hint: Rationalize]
1 1
7. (i) ∑ sin , (ii) ∑ cos
n2 n
1 1
8. ∑ tan
n n
R| n +1 − n −1 U|
9. ∑ S| V|
T n W
1 1 1
10. Show that the series 1 + 2
+ 3 + 4 + ... converges.
2 3 4
[Hint: nn > 2n, for n > 2. Compare with the convergent geometric series ∑1/2 n ]
ANSWERS
1. Convergent, 2. Divergent, 3. Convergent, 4. Convergent for p > 2, 5. Divergent, 6. Convergent,
7. (i) Convergent, (ii) Divergent, 8. Convergent, 9. Convergent.
If ∑ un is a positive term series, such that lim ( un )1/n = l , then the series
n →∞
bu g
n
1/n
- l < A, "n³m
⇒ l - A < un b g 1/n
< l + A, "n³m
⇒ bl - A g < u < bl + A g
n
n
n
= =n, "n³m
⇒ un < = n , " n ³ m.
But since ∑ α n is a convergent geometric series (common ratio α < 1), therefore, by comparison
Copyright © 2004. New Academic Science. All rights reserved.
b g < l + A,
l - A < un
1/n
" n ³ m1
Þ bl - A g < u < bl - A g ,
n n
n " n ³ m1
Þ u > bl - A g = > ,
n n
n " n ³ m1 .
n
But since ∑ β is a divergent geometric series (common ratio β > 1), therefore by comparison
test, the series ∑ un diverges.
∑ (1/n) diverges, while lim (1/n )1/n = 1, and ∑ (1/n 2 ), converges, while lim (1/n 2 )1/n = 1.
n →∞ n →∞
F 1 IJ
Test for convergence of the series whose general term is G 1 +
− n 3/ 2
Example 4.9.
H nK .
1
Solution. Let un = , then
e1 + 1/ n j
n 3 /2
un + 1
If ∑ un is a positive term series, such that lim = l , then the series
n →∞ un
(i) converges, if l < 1,
(ii) diverges, if l > 1, and
(iii) the test fails, if l = 1.
Case I. 0 < l < 1.
Let us select a positive number ε , such that l + ε < 1.
Let l + ε = α < 1, α ≠ 0.
Copyright © 2004. New Academic Science. All rights reserved.
un + 1
Since, lim = l , therefore ∃ a positive integer m such that
n →∞ un
un + 1
− l < ε, ∀ n ≥ m
un
un + 1
⇒ l-A< < l + A, "n³m
un
un + 1
⇒ < l + A = =, "n³m
un
For n ≥ m,
un um + 1 um + 2 u
= ⋅ ... n < α n − m
um um um +1 un − 1
FG u IJ = ,
H= K
m n
⇒ un < m " n ³ m, = < 1.
Thus, ∀ n ≥ m, we have
un < kα n
But since ∑ α n is a convergent geometric series (common ratio, = < 1), therefore by comparison
test ∑ un converges.
Case II. l > 1.
Let us select a positive number ε , such that l − ε > 1.
Let l − ε = β > 1.
un + 1
Since, lim = l , therefore ∃ a positive integer m1 such that
n →∞ un
un + 1
l-A< < l + A, " n ³ m1
un
un + 1
⇒ > l - A = >, " n ³ m1
un
Now, for n ≥ m1 ,
un um + 1 um + 2 u
= 1 × 1 ... n ³ > n - m1
um1 um1 um1 + 1 un - 1
um1
Copyright © 2004. New Academic Science. All rights reserved.
⇒ >n,
un ³ " n ³ m1
> m1
m
Since m1 is a fixed integer, therefore um1 /β 1 is a fixed finite number, say k1.
Thus, for n ≥ m1 , we have
un > k1> n .
n
But ∑ β is a divergent geometric series (common ratio, > > 1); therefore by comparison test,
∑ un diverges.
Note. The test fails for l = 1 in the sense that it fails to give any definite information.
For example, consider the two series ∑(1/n) and ∑ (1/n 2 )
∑
1 un + 1
= lim
n 1 u +1
= 1, and ∑ 2 converges, while lim n = lim
n FG IJ 2
n
diverges, while lim
n →∞ u
n
n →∞ n + 1 n n →∞ u n
n →∞ n +1 H K = 1.
n2 − 1 n
Example 4.10. Test for convergence of the series ∑ x , x > 0.
n2 + 1
n2 − 1 n
Solution. Let un = x .
n2 + 1
lim
un + 1
= lim 2 ⋅ ⋅ n =x
b
n2 + 1 n + 1 − 1 x n + 1 g 2
∴ n →∞ u
n
n →∞ n − 1 2
n+1 +1 x b g
Hence by D’Alembert’s Ratio Test the series converges if x < 1 and diverges if x > 1.
The test fails to give any information when x = 1.
n2 − 1
When x = 1, un = , and lim un = 1 ≠ 0
n2 + 1 n →∞
Fu I
If ∑ un is a positive term series, such that lim n
n →∞
GH u
n
n +1
JK
− 1 = 1, then the series
Let l − ε = α > 1.
Fu I
Since lim n
n →∞
GH u
n
n +1
JK
− 1 = l , therefore ∃ a positive integer m such that for all n ≥ m,
Fu I
l−ε<n GH u n
n +1
JK
−1 <l +ε
Fu I
⇒ n GH u n +1
n
JK
-1 >l -A ==
⇒ b g
nun - n + 1 un + 1 > = - 1 un + 1 , = - 1 > 0 b g
Putting n = m, m + 1, m + 2, ..., n – 1, and adding, we get
b gd
mum − nun > α − 1 um + 1 + um + 2 + ... + un i
n
= (α − 1) ( Sn − Sm ) , where Sn = ∑u r
r =1
⇒ b= - 1g b S n g
- S m < mum , "n³m
m
⇒ Sn < Sm + um , " n ³ m,
= -1
m
Since m is a fixed integer, therefore Sm + um is a fixed finite number.
α −1
Thus, the sequence {Sn} of partial sums of the given series is bounded above and hence the series
∑ un is convergent.
Case II. l < 1.
Let us select a positive number ε , such that l + A < 1.
Fu I
Since lim n
n →∞
GH u
n
n +1
JK
− 1 = l , therefore ∃ a positive integer m such that for all n ≥ m,
Fu I
l−ε<n GH u n
n +1
JK
−1 <l +ε <1
un n +1
⇒ < , "n³m
un + 1 n
un v
< n
un + 1 v n + 1
Hence by comparison test the series ∑ un diverges.
1 1
Notes 1. The test fails to give any definite information for l = 1. Consider the two series ∑ and ∑ .
n b
n log n g 2
Fu I
lim n
n →∞
GH u
n+1
n
JK
− 1 = 1 for both.
2. Raabe’s Test is stronger that D’Alembert’s Ratio Test and may succeed where Ratio Test fails.
Solution. Now
un
= xn
3⋅ 6 ⋅ 9 ... 3n 7 ⋅10 ⋅13 ... 3n + 4 3n + 7 1 b gb g
un + 1 7 ⋅10 ... 3n + 4b ⋅
g
3⋅ 6 ⋅ 9 ... 3n 3n + 3 x n+1
b g
3n + 7 1
= ⋅
3n + 3 x
un + 1
∴ lim =x.
n →∞ un
By Ratio Test, the series converges for x < 1 and diverges for x > 1. The test fails for x = 1.
But for x = 1,
un 3n + 7
= .
un + 1 3n + 3
Fu I
∴ lim n
n →∞
GH u n +1
n
JK
− 1 = lim
n→∞
4n 4
= >1
3n + 3 3
EXERCISE
Test the behaviour of the following series:
1 1 1 1
1. + + + + ...
2 2 ⋅ 22 3 ⋅ 23 4 ⋅ 24
2.
H 3K H 3⋅ 5 K H 3⋅ 5 ⋅ 7 K + ...
x x2 x3 x4
3. + + + + ...,
1 2 3 4
x2 x3 x4 x5
4. + + + + ...,
2 1 3 2 4 3 5 4
x2 x4 x6
5. 1 + + + + ...
2p 4p 6p
12 ⋅ 2 2 2 2 ⋅ 32 32 ⋅ 42 4 2 ⋅52
6. + + + + ...
1! 2! 3! 4!
F2 I + F3 I + F4 I
Copyright © 2004. New Academic Science. All rights reserved.
2 −1 3 −2 4 −3
7. GH 1
2
−
2
1 JK GH 3 3
−
3
2 JK GH 3 4
−
4
3 JK + ...
1 1 1 1
8. + + 3 + 4 + ...
2 32 2 3
Test for convergence each of the following series whose nth terms are given:
n
9. ,r>0
nn
1⋅ 2 ⋅ 3 ... n
b
10. 7 ⋅ 10 ... 3n + 4
g
n
r
11. , r >0
nn
n xn
12. , x>0
n2 + 1
13.
b
1⋅ 3⋅5 ... 2n − 1 g⋅1
2 ⋅ 4 ⋅ 6 ... 2n n
b g⋅x
1⋅ 3⋅ 5 ... 4n − 3 2n
2 ⋅ 4 ⋅ 6 ... b4n − 2g 4n
14.
n −1 n
15. x , x>0
n3 + 1
b
2 × 4 × 6 ... 2 n + 2 g
b gx
n-1
16.
3× 5 × 7 ... 2n + 3
ANSWERS
1. Converges 2. Converges
3. Converges for x < 1, diverges for x≥1 4. Converges for x ≤ 1, diverges for x > 1
5. Converges for | x | < 1, diverges for | x | > 1; for x = 1, converges for p > 1, diverges for p ≤1
6. Convergent 7. Convergent
8. Convergent 9. Convergent
10. Convergent 11. Convergent
12. Convergent for x < 1, divergent for x ≥ 1 13. Convergent
14. Convergent for x £ 1, divergent for x > 1 15. Convergent for x < 1, divergent for x ≥ 1
F I
n ®¥
GH
lim n log
un
JK
un + 1
= l,
then the series converges for l > 1, and diverges for l < 1.
FG1 + 1 IJ n
H nK ≤ e, ∀ n
so that, we get
un FG
1 IJ = bn + 1g
α α
vn
un + 1
> 1+
H
n K n α
=
vn + 1
, ∀n≥m
where vn = 1/nα .
But since for α > 1, ∑ vn converges, therefore by comparison test, ∑ un also converges.
We may similarly show that for l < 1, the series ∑ un diverges.
Example 4.13. Test for convergence the series
x 2 2 x 2 33 x 3
1+ + + + ..., for x > 0
1! 2! 3!
nn x n
Solution. Ignoring the first term, un = .
n!
un + 1 FG n + 1IJ n
Copyright © 2004. New Academic Science. All rights reserved.
Now lim
n →∞ un
= lim
n→∞ H n K x = ex
By Ratio Test, the series converges for ex < 1 or x < 1/e, and diverges for x > 1/e.
For x = 1/e,
un n FG IJ n
un + 1
=
n+1 H K ⋅e
Now
F I LM FG IJ OP
GH
lim n log
n →∞
un
un + 1 n
JK
= lim n 1 − n log 1 +
→∞ N
1
n
.
H KQ
L F1 1
= lim n M1 − n G −
1 IJ OP = 1 < 1
n →∞ N H n 2n 2
+
3n 3
− ...
KQ 2
Therefore by logarithmic test, the series diverges.
Hence, the series converges for x < 1/e, and diverges for x ≥ 1/e.
Note. Logarithmic test is generally more helpful in situations like those above, where the presence of a number like e
in un/un + 1 makes the application of Raabe’s Test difficult.
Further, if u( x) ≥ 0 for all x > a, it can be shown geometrically or otherwise that the integral
z t
u( x ) dx is a monotonic increasing function of t, so that the improper integral
z
a
∞
u( x ) dx , where u ( x ) ≥ 0, ∀ x > a
a
converges iff it is bounded above i.e., ∃ a positive number k such that
z a
t
u( t ) dt ≤ k , ∀ t ≥ a
z ∞
u( x ) dx converge or diverge together.
Copyright © 2004. New Academic Science. All rights reserved.
n =1 1
z n
n +1
u(n) dx ³ zn
n+1
u( x ) dx ³ z
n
n+1
u( n + 1) dx
⇒ bg
un ≥ z
n
n +1
u( x ) dx ≥ u n + 1 b g
or un ≥ zn
n +1
u( x ) dx ≥ un + 1 . ...(1)
S n − un ≥ I n ≥ S n − u1
⇒ 0 < un £ Sn - I n £ u1 ...(2)
Let us consider the sequence {(Sn – In)}.
bS − I g − dS
n n n −1 − i
I n − 1 = Sn − Sn − 1 − I n − I n − 1 d i
= un − z
n −1
n
u( x ) dx
≤0 [using (1)].
Therefore, the sequence {(Sn – In)} is monotonic decreasing, bounded by 0 and u1.
Hence, the sequence converges and has a limit such that
b
0 ≤ lim Sn − I n ≤ u1 g ...(3)
the series differs from the integral by less than u1; if divergent, the limit of (Sn – In) still exists and lies
z ∞
1
u( x ) dx; if convergent, the sum of
Example 4.14. Show that the series ∑(1/n p ) converges, if p > 1, and diverges if p ≤ 1.
Solution. Let u(x) = 1/xp, so that for x ≥ 1, the function u is a non-negative monotonic decreasing
integrable function such that
un = u n = bg 1
np
, ∀ n ∈N
n =1 z
1
∞
u( x ) dx converge or diverge together.
x x 1
∵ ∫1 u ( x)dx = ∫1 xp
dx = 1 − p
log x, if p = 1
1 if p > 1
∞ x ,
∴ ∫1 u ( x )dx = lim ∫
x →∞ 1
u ( x) dx = p − 1
∞, if 0 < p ≤ 1
Thus z1
∞
u( x ) dx converges, if p > 1, and diverges, if 0 < p ≤ 1.
Hence, the infinite series ∑(1/n p ) converges if p > 1, and diverges if 0 < p ≤ 1.
But, when p < 0, the series ∑(1/n p ) diverges for then the nth term n–p does not tend to zero as
n → ∞.
Hence the series ∑(1/n p ) converges when p > 1, and diverges when p ≤ 1.
∞ 1
Example 4.15. The series ∑ , p > 0, converges for p > 1 and diverges for p ≤ 1.
n=2 b
n log n g p
1
Solution. Let u( x ) = , so that for x ≥ 2 , the function u is a non-negative monotonic decreasing
b
x log x g p
un = u n = b g nb log1 ng p
, ∀ p > 0, n ∈ N
3 z X
2
u( x ) dx = zX
2
b
x log x
1
g p
dx , p>0
R| blog X g − blog 2g 1− p 1− p
=S , if p ≠ 1,
|Tlog log X1−−logp log 2, if p = 1.
R| blog 2g
z z
1− p
u( x ) dx = S p − 1
∞ X
u( x ) dx = lim , if p > 1,
∴
2 X →∞ 2
|T ∞ , if 0 < p ≤ 1.
Thus z2
∞
u( x ) dx converges if p > 1, and diverges if 0 < p ≤ 1.
∞ 1
Hence, the series ∑ , p > 0, converges if p > 1, and diverges if p ≤ 1.
b g p
Copyright © 2004. New Academic Science. All rights reserved.
n=2 n log n
Hence by Raabe’s Test, the series converges if β > 1, and diverges if β < 1.
For β = 1, we have
un 1 γ
= 1 + + np , p>1
un + 1 n n
1
Let us compare the given series with the divergent series ∑ vn where vn = .
n log n
Now
un v 1 γ
− n = 1 + + np −
n + 1 log n + 1 b g b g
un + 1 vn + 1 n n n log n
= −
LM b g OP
γ n n + 1 log n + 1
−1
np n Nlog n Q
1 L F 1 I n OP
γ − bn + 1g log G1 + J ⋅
p −1
=
n N
Mp n
H n K log n Q
F 1I L F 1I F 1I O
lim bn + 1g log G 1 + J = lim Mlog G1 + J + log G 1 + J P = 1
n
Since
n=∞ H nK n →∞
MN H n K H n K PQ
np −1
Copyright © 2004. New Academic Science. All rights reserved.
and nlim = ∞ , p > 1, and {γ n } is bounded, therefore, for sufficiently large values of n,
=∞ log n
b g FGH
γ n − n + 1 log 1 +
IJ
1 np−1
K
⋅
n log n
remains negative.
b g FGH
γ n − n + 1 log 1 + ⋅
IJ
1 np −1
K
n log n
< 0, ∀n≥m
un v
⇒ − n < 0, ∀n≥m
un + 1 v n + 1
un vn ,
⇒ < ∀n≥m
un + 1 vn + 1
Remarks 1. Gauss’s Test is very useful and may be used after the failure of Raabe’s Test or directly without recourse
to other tests.
2. If
un β γ δ
= α + + 2 + 3 + ... ...(1)
un + 1 n n n
where α , β , γ ... are independent of n, then we can write
un β γ
= α + + 2n
un + 1 n n
where γ n = γ + δ /n + ... , so that lim γ n = γ , i.e., {γ n } is a bounded sequence.
Thus for the application of Gauss’s Test we may expand un /un + 1 in powers of 1/n as in (1).
2 2 2 2 ⋅ 4 2 2 2 ⋅ 4 2 ⋅ 62
+ 2 2 + 2 2 2 + ...
32 3 ⋅5 3 ⋅5 ⋅7
Solution. Now
un =
2 2 ⋅ 4 2 ⋅ 62 ... 2 nb g 2
b
32 ⋅52 ⋅ 7 2 ... 2n + 1 g 2
un b g = FG1 + 3 IJ FG1 + 1 IJ
2n + 3
2 2 −2
b g H 2n K H n K
∴ = 2
un + 1 2n + 2
F 3 9 IJ FG1 − 2 + 3 − ...IJ
= G1 + +
Copyright © 2004. New Academic Science. All rights reserved.
H n 4n K H n n K 2 2
1 3 1
=1+ − 2 + ... higher powers of ,
n 4n n
so that α = 1 and β = 1.
Hence by Gauss’s Test, the series diverges.
∑
b
12 ⋅ 32 ... 2n − 1 g 2
xn − 1 , x > 0.
2 2
2 ⋅ 4 ... 2n b g 2
Solution. Here
u
lim n = lim
2n + 2 1 1
⋅ =
b g 2
n →∞ u
n+1
n→∞ 2 + 1
n
2
x x b g
Hence by Ratio Test, the series converges if x < 1, and diverges if x > 1
Now for x = 1,
un
=
2n + 2 b g 2
un + 1 2n + 1
2
b g
Fu I 4 n 2 + 3n
∴ lim n
n →∞
GH u n
n +1
JK
− 1 = lim
n →∞
b2n + 1g 2
=1
un FG IJ FG1 + 1 IJ
1
2 −2
un + 1 H K H 2n K
= 1+
n
F 2 1 IJ FG1 − 1 + 3
= G1 + +
IJ
H n n K H n 4n 2 2
+ ...
K
1 1 1
− =1+ + ... higher powers of
n 4n 2 n
so that by Gauss’s Test, the series diverges.
Hence, the series converges for x < 1, and diverges for x ≥ 1.
un FG
1 2n + 2 IJ 2
1 1/ x 1/4 x
un + 1
=
H
x 2n + 1 K =
x
+
n
− 2 + ...
n
un b
n+1 γ +n 1 1 gb g
lim
n →∞
= lim
b =
un + 1 n → ∞ α + n β + n x x gb g
Hence, by Ratio Test, the series converges, if x < 1, and diverges, if x > 1.
For x = 1, we have
FG1 + 1 IJ FG1 + γ IJ FG1 + γ + 1 + γ IJ
un bn + 1gbγ + ng = H n K H n K = H n n K 2
un + 1
=
bα + ngbβ + ng FG1 + α IJ FG1 + β IJ FG1 + α + β + αβ IJ
H nK H nK H n n K 2
F γ + 1 + γ IJ LM1 − FG α + β + αβ IJ + FG α + β + αβ IJ + ...OP
= G1 +
2
H n n K MN H n n K H n n K PQ
2 2 2
1 + γ − α − β bα + β − γ gbα + β − 1g − αβ
=1+ + + ...
n n2
Hence by Gauss’s Test the series converges if 1 + γ − α − β > 1 or γ > α + β and diverges if
1 + γ − α − β ≤ 1 or γ ≤ α + β .
Thus for positive values of α , β , γ and x,
(i) for x < 1, the series converges,
(ii) for x > 1, the series diverges, and
(iii) for x = 1, the series converges if γ > (α + β ) and diverges if γ ≤ (α + β ).
EXERCISE
Test the convergence of the series:
2 x 32 ⋅ x 2 43 ⋅ x 3
1. 1 + + + + ...
2! 3! 4!
Copyright © 2004. New Academic Science. All rights reserved.
x 1 x3 1× 3 x 5 1× 3× 5 x 7
2. + + + + ...
1 2 3 2 ×4 5 2 ×4 × 6 7
3.
a+x
+
b
a + 2x g + ba + 3xg
2 3
bn!g x
b2ng!
n
7.
n!
8. xn
bn + 1g n
b
α α α +1 g
α α +1 α +2 b gb g
9. Prove that 1 + +
b
β β β +1
+
g
β β +1 β + 2b gb g
+ ..., where x, are positive, converges if β > α + 1, and
diverges if β ≤ α + 1.
ANSWERS
1. Converges if x < 1/e, diverges if x ≥ 1/e .
2. Converges if x £ 1, diverges if | x | > 1.
3. Converges for x < 1/e, diverges for x ≥ 1/e .
4. Diverges
5. (i) Convergent (ii) Convergent.
6. Converges for p > 1, diverges for p ≤ 1.
7. Converges for x < 4, diverges for x ≥ 4.
8. Converges for 0 < x < e, diverges for x ≥ e .
2 3 4
is called an alternating series.
Leibnitz Test. If the alternating series
b
u1 − u2 + u3 − u4 + ..., un > 0, ∀ ng
is such that
(i) un + 1 ≤ un , ∀ n , and
We shall now show that the sequence {S2n + 1} also converges to the same limit S.
Now
S2n + 1 = S2n + u2 n + 1
Thus, the sequences {S2n} and {S2n + 1} both converge to the same limit S. We shall now show that
the sequence {Sn} also converges to S.
Let ε > 0 be given.
Since the sequences {S2n} and {S2n + 1} both converge to S, therefore ∃ positive integers m1, m2,
respectively, such that
Copyright © 2004. New Academic Science. All rights reserved.
S2 n − S < ε , ∀ n ≥ m1 ...(1)
and
S2 n + 1 − S < ε , ∀ n ≥ m2 ...(2)
1 1 1 1
Example 4.19. Show that the series p − p + p − p + ... converges for p > 0.
1 2 3 4
Solution. Let un = 1/np.
Here
un + 1 ≤ un ; ∀ n
and
1
lim un = lim =0
n →∞ n →∞ np
( −1) n − 1
Hence by Leibnitz test, the alternating series ∑ converges.
np
Conditional Convergence. A series which is convergent but is not absolutely convergent is called a
conditionally convergent series.
ILLUSTRATIONS
1. The series
1 1 1
1− 3
+ 3 − 3 + ... ,
2 3 4
is absolutely convergent because the series
1 1 1
1 + 3 + 3 + 3 + ... ,
2 3 4
obtained on taking every term of the given series with a positive sign, is convergent.
2. The series
Copyright © 2004. New Academic Science. All rights reserved.
1 1 1
1 - + - + ...
2 3 4
is convergent by Leibnitz test, but the series
1 1 1
1+ + + + ...
2 3 4
obtained on taking every term with a positive sign is divergent. Thus the given series is
conditionally convergent.
Let an =
RSu , if u ≥ 0
n n
and bn =
RS −u , if u < 0
n n
T 0 , if u < 0
n T0 , if u ≥ 0 .
n
Then clearly,
an ≥ 0 , bn ≥ 0 , ...(i)
un = an − bn ...(ii)
and
un = an + bn ...(iii)
From (i) and (iii) it follows that
an £ un , bn £ un
Since ∑ un is convergent, therefore, by Comparison Test, both ∑ an and ∑ bn are convergent.
(-1) n - 1
For example, if un = , we have seen above that ∑ un is divergent, whereas ∑ un is convergent.
n
2. The very great significance of the concept of Absolute Convergence is that the convergence of absolutely
convergent series is much more easy to recognise than that of conditionally convergent series—usually by
comparison with series of positive terms. In fact all the tests for positive term series become available for
Copyright © 2004. New Academic Science. All rights reserved.
the purpose. But this significance becomes all the more visible in the discussion of rearrangement of
series—so much so that we may operate on absolutely convergent series, precisely as we operate on sums
of a finite number of terms, whereas in the case of conditionally convergent series this in general is not
possible.
∞ sin nx
Example 4.20. Show that for any fixed value of x, the series ∑ is convergent.
n =1 n2
sin nx sin nx
Solution. Let un = 2
, so that un =
n n2
sin nx 1 1
Now 2
≤ 2 , ∀ n and ∑ 2 converges.
n n n
sin nx
Hence, by Comparison Test, the series ∑ converges.
n2
sin nx
Since every absolutely convergent series is convergent, therefore Σ is convergent.
n2
Example 4.21. Show that the series
x2 x3
x+ + + ...
2! 3!
converges absolutely for all values of x.
xn
Solution. Let un = .
n!
un n+1
Now lim = lim → ∞ except when x = 0.
n →∞ un + 1 n→∞ x
So by Ratio Test, the series converges absolutely for all x except possibly zero.
But for x = 0 the series evidently converges absolutely.
Hence, the series converges absolutely for all values of x.
xn
∴ lim = 0 a useful result.
n →∞ n!
b g b
m m − 1 ... m − n + 1 gx n
un =
bn − 1g!
Now
un n 1
lim = lim ⋅
n →∞ un + 1 n →∞ m−n x
1 1 1
= lim ⋅ =
m n →∞ x x
−1
n
Hence, the series ∑ un converges absolutely for x < 1
⇒ the series ∑ un converges for x < 1
b g b
m m − 1 ... m − n + 1 gx
bn − 1g!
n
i.e., lim = 0 , if x < 1
n →∞
Note. The results of Example 4.21 and 4.22 are very useful.
EXERCISE
1. Show that the following series are convergent:
1 1 1
(i) 1 − + − + ...
2 3 4
1 1 1 1
(ii) − + 2 − 2 + ...
12 2 2 3 4
1 1 1
(iii) 1 − + − + ...
2 2 3 3 4 4
log 2 log 3 log 4
(iv) − 2 + − ...
22 3 42
2. Prove that the following series are absolutely convergent:
x x2
(i) 1 + + + ...
1! 2!
x2 x4 x6
Copyright © 2004. New Academic Science. All rights reserved.
(ii) 1 − + − + ...
2! 4! 6!
x 3 x5
(iii) x − + − ...
3! 5!
nr
3. Show that lim =0
n ®¥ xn
(i) ∑
b−1g n +1
(ii) ∑
b−1g n +1
3n − 2
( −1) n + 1
5. Show that the series ∑ is absolutely convergent for p > 1, but conditionally convergent for 0 < p ≤ 1.
np
6. Show that the following series are absolutely convergent:
R U
(i) b g |S| n1 + bn +1 1g |V|
∑ −1
n −1
T 2
W 2
R| 1 U
(ii) ∑ b −1g S| n + bn +11g |V|
n−1
T 5 /2
W 5 /2
(iii) b g
∑ −1
n n+2
2n + 5
F 1 + b−1g I is divergent.
n −1
7. Show that the series ∑ GH n n JK
1 1 1
8. Show that the series 1 − + − + ... converges.
3⋅ 4 5⋅ 4 2 7 ⋅43
Copyright © 2004. New Academic Science. All rights reserved.